Đến nội dung

JUV nội dung

Có 136 mục bởi JUV (Tìm giới hạn từ 30-03-2020)



Sắp theo                Sắp xếp  

#694374 Tuần $2/10$ năm $2017$: Tâm $(PBC)$ nằm trên...

Đã gửi bởi JUV on 08-10-2017 - 18:38 trong Chuyên mục Mỗi tuần một bài toán Hình học

Như vậy lời giải cho hai bài Tuần 1, tháng 10/2017 đã được đưa tại đây kèm theo đó là hai bài toán mới của thầy Trần Quang Hùng và thầy Nguyễn Tiến Dũng. Xin được trích dẫn lại hai bài toán:

Bài 1:  Cho tam giác $ABC$ có tâm nội tiếp $I$, phân giác $AD$. $K,L$ là tâm nội tiếp $ABD,ACD$.$J$ là tâm $(AKL)$.$IJ$ cắt $(IKL)$ tại $P$ khác $I$.Chứng minh tâm $(PBC)$ nằm trên $(O)$

Hình vẽ:

D1IjxvU.png

Bài 2: Cho tam giác $ABC$ nội tiếp $(O)$. $D,E$ thuộc $CA,AB$ sao cho $O$ là trung điểm $DE$ và $DE=OA$.$K$ đối xứng $O$ qua $BC$. Lấy $M,N$ để $OM,ON$ lần lượt song song $CA,AB$, $K$ là trung điểm $MN$. $BN$ cắt $CM$ tạp $P$. Chứng minh $(PMN)$ tiếp xúc $(O)$

Hình vẽ:

uM3GYxt.png

 

 



#693654 Tuần $4$ tháng $9/2017$: $AP$ đi qua điểm cố định

Đã gửi bởi JUV on 24-09-2017 - 19:45 trong Chuyên mục Mỗi tuần một bài toán Hình học

Như vậy lời giải cho hai bài Tuần 3 tháng 9/2017 đã được đưa tại đây kèm theo đó là hai bài toán mới của thầy Trần Quang Hùng và thầy Nguyễn Tiến Dũng. Xin được trích dẫn lại hai bài toán:

Bài 1: Cho tam giác $ABC$ nội tiếp đường tròn $(O)$ với $B,C$ cố định và $A$ thay đổi trên $(O)$. $I$ là tâm nội tiếp, $BI,CI$ cắt lại $(O)$ tại $E,F$. Lấy $M,N$ để $AM,EM,AN,FN$ lần lượt vuông góc $AF,CF,AE,BE$. Đường qua trung điểm $AI$ song song $BC$ cắt $AB,AC$ tại $R,Q$. $K,L$ là hình chiếu của $A$ lên $FM,EN$. $QL$ cắt $RK$ tại $P$. Chứng minh $AP$ luôn đi qua điểm cố định khi $A$ thay đổi

Hình vẽ
B3tAIUU.png
Bài 2: Cho $ABC$ nội tiếp đường tròn $(O)$ có $2$ điểm Brocard $\Omega_1, \Omega_2$. Chứng minh nếu có $1$ trong $6$ góc $\angle A\Omega_1 O,\angle B\Omega_1O, \angle C\Omega_1O,A\Omega_2 O,\angle B\Omega_2O, \angle C\Omega_2O$ vuông thì có đúng $2$ trong $6$ góc là vuông
Hình vẽ:
4i3Hy4z.png



#692818 Tuần $2$ tháng $9/2017$: Chứng minh $\frac...

Đã gửi bởi JUV on 10-09-2017 - 22:33 trong Chuyên mục Mỗi tuần một bài toán Hình học

Như vậy lời giải cho hai bài Tuần 1, tháng 9, 2017 đã được đưa tại đây kèm theo đó là hai bài toán mới của thầy Trần Quang Hùng và anh Ngô Quang Dương. Xin được trích dẫn lại hai bài toán:

Bài 1: Cho tam giác $ABC$ và $M,N$ nằm trên cạnh $BC$ sao cho $M$ nằm giữa $N,B$.Lấy $P,Q$ trên $AM,AN$ để $BP,CQ$ cùng vuông góc với $BC$. $K,J$ là tâm ngoại tiếp $(APQ),(AMN)$. $L$ là hình chiếu của $K$ lên $AJ$. Chứng minh $\frac{AJ}{AL}=\frac{MN}{BC}$

Hình vẽ

eM2iXER.png

Bài 2: Cho tam giác $ABC$ và $l$ là 1 đường thẳng bất kì. $D,E,F$ lần lượt là hình chiếu của $A,B,C$ lên $l$.$X,Y,Z$ lần lượt chia $AD,BE,CF$ theo cùng $1$ tỉ số $k$. Các đường lần lượt qua $X,Y,Z$ và vuông góc $BC,CA,AB$ đồng quy tại $K$. Chứng minh $(KAX),(KBY),(KCZ)$ đồng trục và trục đẳng phương của chúng đi qua điểm cố định khi $k$ thay đổi.

Hình vẽ

z6aGTL9.png




#692322 Chứng minh tồn tại số $n$ thỏa mãn 2 điều kiện

Đã gửi bởi JUV on 04-09-2017 - 14:53 trong Số học

$n$ là số có dạng $999...9$ sao cho $n$ có nhiều chữ số hơn $m$




#691756 Tuần 5 tháng 8/2017: $YC=ZB$

Đã gửi bởi JUV on 28-08-2017 - 23:01 trong Chuyên mục Mỗi tuần một bài toán Hình học

Bài 1:  Gọi $O_1$,$O_2$ là tâm đường tròn $CMZ$ và $BNY$. Dễ thấy $\angle CO_1Z=\angle BO_2Y=\alpha = \pi -\angle A$ (do $\angle CMZ=\angle BNY= \frac{\pi+\angle A}{2}$) và $CO_1Z$ và $BO_2Y$ cân tại $O_1,O_2$ nên $\exists k\in R$ sao cho $O_1\in f_1(AB), O_2\in f_2(AC)$ với $f_1$ là phép đồng dạng quay với tâm $C$, hệ số đồng dạng $k$, hệ số góc $\beta $ và $f_2$ là phép đồng dạng quay tâm $B$, hệ số đồng dạng $k$, hệ số góc $-\beta$ với $\beta=\frac{\pi -\alpha }{2}$. Dễ thấy $f_1(AB)=f_2(AC)=AD$ và $f_1(B)=f_2(C)=I$ là điểm chính giữa cung $BC$ không chứa $A$ của $(ABC)$ . Gọi $K_1$,$K_2$ là trung điểm $DC$,$DB$. Ta có $O_1M_1$ đi qua trung điểm $CE$ (do $O_1M_1$ là trung trực $MC$), tương tự $O_2M_2$ đi qua trung điểm $BF$ và $O_1,O_2$ đều thuộc $AD$ nên theo định lý $Menelaus$, $O_1\equiv O_2\equiv O$. Vậy $BZ=\frac{OI}{k}=CY$




#691712 Tuần 5 tháng 8/2017: $YC=ZB$

Đã gửi bởi JUV on 27-08-2017 - 22:13 trong Chuyên mục Mỗi tuần một bài toán Hình học

Như vậy lời giải cho hai bài Tuần 4 tháng 8/2017 đã được đưa tại đây kèm theo đó là hai bài toán mới của thầy Trần Quang Hùng và bạn Đỗ Xuân Long. Xin được trích dẫn lại hai bài toán:

Bài 1:  Cho tam giác $ABC$ có phân giác $AD$.$E,F$ lần lượt thuộc $CA,AB$ sao cho $EF$ song song $BC$. Gọi $M,N$ theo thứ tự là hình chiếu của $C,B$ lên $DE,DF$. $AD$ cắt đường tròn ngoại tiếp các tam giác $AEM$ và $AFN$ tại lần lượt tại $U,V$ khác $A$. Gọi $NV,MU$ lần lượt cắt $CA,AB$ tại $Y,Z$ Chứng minh rằng $YC=ZB$

Hình vẽ:

etXvnkZ.png

Bài 2: Cho lục giác $ABCDE$ nội tiếp có $AB=CD=EF$ và $BC=DE$. $P$ là một điểm di chuyển trên cung nhỏ $AF$ của đường tròn ngoại tiếp lục giác. $PC,PD$ lần lượt cắt $AE,FB$ tại $M,N$. $K,L$ theo thứ tự thuộc các cạnh $AB,EF$ sao cho $MK,NL,AF$ đôi một song song. $PC,PD$ lần lượt cắt $AF$ tại $S,T$. $KS$ cắt $LT$ tại $Q$. Chứng minh rằng đường thẳng $PQ$ chia đôi đoạn $CD$

Hình vẽ

nFt2DRJ.png




#691332 Cho 12 số nguyên tố phân biệt

Đã gửi bởi JUV on 23-08-2017 - 18:07 trong Tổ hợp và rời rạc

Câu 2: Có $2000>e \times 6! \geq R(3;3;3;3;3;3)$ nên theo định lý Schur $\Rightarrow$ $Q.E.D$




#690469 Tuần 3 tháng 8/2017: $PQ$ chia đôi $CD$

Đã gửi bởi JUV on 13-08-2017 - 21:58 trong Chuyên mục Mỗi tuần một bài toán Hình học

Như vậy lời giải cho hai bài Tuần 2 tháng 8/2017 đã được đưa tại đây kèm theo đó là hai bài toán mới của thầy Trần Quang Hùng và bạn Đỗ Xuân Long. Xin được trích dẫn lại hai bài toán:

Bài 1: Cho lục giác $ABCDE$ nội tiếp có $AB=CD=EF$ và $BC=DE$.$P$ di chuyển trên cung nhỏ $AF$ của đường tròn ngoại tiếp lục giác. $PC,PD$ lần lượt cắt $AE,BF$ tại $M,N$.$K,L$ theo thứ tự là hình chiếu của $M,N$ lên cạnh $AF$. $ML$ cắt $NK$ tại $Q$. Chứng minh đường thẳng $PQ$ chia đôi $CD$

Hình vẽ:

ko7ppyI.png
Bài 2: Cho tam giác $ABC$ nhọn, $1$ đường tròn $(K)$ đi qua $B,C$ lần lượt cắt $CA,AB$ tại $E,F$. $(I),(J)$ tiếp xúc $AR$ tại $R$ và tiếp xúc trong với $(K)$ theo thứ tự tại $M,N$ sao cho $I,J$ đều nằm trong các góc 
$\angle FRB$ và $\angle ERC$. Chứng minh $ME,NF$ cắt nhau trên đường thẳng $AR$

Hình vẽ: 

fhLYh0E.png




#690215 Cho 1251 số

Đã gửi bởi JUV on 11-08-2017 - 12:40 trong Tổ hợp và rời rạc

Giả sử $\left | \sqrt{ma_{m}} -\sqrt{na_{n}}\right | \leq 5$ $\forall m,n= \overline{1,2014}, m \neq n$. Lúc đó sẽ tồn tại $a \in \mathbb{R}$ để $a \leq \sqrt{ma_{m}} \leq a+5$ $\forall m= \overline{1,2014}$.Vì vậy tồn tại $c \in \mathbb{N}$ để $c \leq \sqrt{ma_{m}} \leq c+6$ $\forall m= \overline{1,2014}$, hay $c^2 \leq ma_m \leq (c+6)^2$. Ta có $2014 \geq a_1 \geq c^2$ và $(c+6)^2 \geq max\left ( 1251a_{1251},1250a_{1250},1249a_{1249} \right )\geq 1249\times 3= 3747$. Dễ thấy không tồn tại $c$ thỏa mãn nên giả sử sai $\Rightarrow Q.E.D$




#689100 Tuần 1 tháng 8/2017: $AU,BV,CW$ đồng quy

Đã gửi bởi JUV on 30-07-2017 - 19:45 trong Chuyên mục Mỗi tuần một bài toán Hình học

Như vậy lời giải cho hai bài Tuần 3 tháng 7/2017 đã được đưa tại đây kèm theo đó là hai bài toán mới của thầy Trần Quang Hùng và thầy Nguyễn Minh Hà. Xin được trích dẫn lại hai bài toán:

Bài 1: (Thầy Trần Quang Hùng) Cho tam giác $ABC$ có điểm $Lemoine$ $L$.$X,Y,Z$ lần lượt nằm trên $LA,LB,LC$ sao cho $YZ,ZX,XY$ lần lượt song song với $BC,CA,AB$. $BZ$ cắt $CY$ tại $D$, $CX$ cắt $AZ$ tại $E$, $AY$ căt $BX$ tại $F$. $U,V,W$ lần lượt đẳng giác với $D,E,F$ trong $LBC,LCA,LAB$. Chứng minh $AU,BV,CW$ đồng quy.

Hình vẽ:  

SUp0Qp8.png

Bài 2: (Thầy Nguyễn Minh Hà) Cho tam giác $ABC$ không đều, $L$ là điểm $Lemoine$. Đường đối trung từ $L$ của $LBC,LCA,LAB$ theo thứ tự cắt lại $(LBC),(LCA),(LAB)$ tại $D,E,F$, Chứng minh $AD,BE,CF$ đồng quy tại $1$ điểm thuộc đường thẳng $Euler$ của $ABC$

Hình vẽ : IjWwPqP.png




#687667 Maryam Mirzakhani đã qua đời

Đã gửi bởi JUV on 16-07-2017 - 07:56 trong Tin tức - Vấn đề - Sự kiện

Đáng buồn thay những thiên tài yểu mệnh !



#687155 Chứng minh rằng: $m\leq C_{n-1}^{k-1}.$

Đã gửi bởi JUV on 10-07-2017 - 19:26 trong Tổ hợp và rời rạc

Quy nạp theo $n$. Dễ thấy với $n=1,2,3$ thì bài toán đúng.
Gọi $A$ là tập các tập con $k$ phần tử của $S$ chứa $n$, $B$ là 1 tập các tập con $k$ phần tử của $S$ mà không chứa $n$ để $2$ tập bất kì trong $B$ có giao khác rỗng.
Lập một đồ thị $2$ phe $A$ và $B$ mà $2$ đỉnh là $2$ tập thuộc $2$ phe nối với nhau nếu giao của chúng bằng rỗng. Ý tưởng là ta sẽ xây dựng $F$ là hợp tập $B$ và $1$ số phần tử tập $A$
Mỗi phần tử tập $B$ nối với đúng $\binom {n-k-1}{k-1}$ phần tử tập $A$ và mỗi phần tử tập $A$ nối với $1$ số tập con của tập có $n-1-(k-1)=n-k$ phần tử, mỗi tập có $k$ phần tử và không có $2$ trong số các tập con đó giao bằng rỗng. Theo quy nạp thì mỗi phần tử tập $A$ nối không quá $\binom{n-k-1}{k-1}$ phần tử tập $B$. Vì vậy số phần tử tập $A$ nối với $1$ trong số các phần tử tập $B$ $\geq |B|$. Khi thêm tập $B$ vào tập $A$ thì phải loại bỏ các phần tử đó từ tập $A$. Vậy $m=|F| \leq |A|+|B|-|B|=|A|=\binom{n-1}{k-1}$



#685179 Đề thi toán chuyên - chuyên KHTN ĐHQG HÀ Nội vòng 2 2017

Đã gửi bởi JUV on 20-06-2017 - 19:37 trong Tài liệu - Đề thi

Câu 4: Chia đa giác thành $k$ miền ngũ giác lồi thì sẽ có tổng $5k$ cạnh. Mỗi cạnh của $n$ giác ban đầu được đếm $1$ lần, các cạnh khác được đếm $2$ lần nên $5k+n$ phải là số chẵn. Do vậy ý b thì câu trả lời là không, còn ý a có thể dễ dàng chỉ ra cách chia thoả mãn




#682747 Basic Matrix

Đã gửi bởi JUV on 02-06-2017 - 13:15 trong Tổ hợp và rời rạc

Cảm ơn anh vì cách làm bài trên, em giải bằng phép truy hồi cũng ra kết quả như thế, ngoài ra thầy em nói là 1 cách có thể dùng định lý Hall @@

Thật ra bổ đề trên phải trừ TH $(k-1)-clique$ nhưng cũng dễ dàng CM mọi đồ thị khác đều thoả mãn bổ đề




#682656 Basic Matrix

Đã gửi bởi JUV on 01-06-2017 - 20:06 trong Tổ hợp và rời rạc

Lemma: Một đồ thị $G$ không có $k-clique$ hay $k-clique$ thiếu $1$ cạnh thì tồn tại đồ thị $G'$ $k-2$ phe cùng tập đỉnh với $G$ có số cạnh không ít hơn $G$

Sketch: Quy nạp theo $k$. Chọn $A$ là điểm có bậc lớn nhất là $k$, $M(A)$ là các đỉnh nối với $A$, $N(A)$ là các đỉnh còn lại. Xét đồ thị sinh bởi $M(A)$ không có $(k-1)-clique$ hay $(k-1)-clique$ thiếu $1$ cạnh nên có đồ thị $H$ $k-3$ phe cùng tập đỉnh và có số cạnh không ít hơn $M(A)$. Bỏ đi tất cả các cạnh trong $N(A)$ và nối tất cả các điểm của $N(A)$ với tất cả các điểm của $H$ đôi một, ta được đồ thị $G'$ $k-2$ phe (Các đỉnh của $N(A)$ là $1$ phe). Bậc của các đỉnh của $N(A)$ trong $G$ đều nhỏ hơn hoặc bằng $k$, còn trong $G'$ thì đúng bằng $k$. Mà $H$ có số cạnh ít nhất bằng $M(A)$ nên $G'$ có số cạnh ít nhất bằng $G$, $Q.E.D$

Thay $k=5$, $G$ có $9$ đỉnh, số cạnh lớn nhất thoả mãn không có $5-clique$ hay $5-clique$ thiếu $1$ cạnh thu được khi đồ thị là đồ thị $3$ phe. Ta dễ thấy số cạnh nhiều nhất là $27$, do vậy có inhiều nhất $27$ cặp điểm không được nối, hay ít nhất $9$ cặp cạnh được nối để thoả mãn đề bài, xảy ra trong trường hợp chia thành $3$ tam giác rời nhau




#678245 USAMO 2017 ngày 1

Đã gửi bởi JUV on 21-04-2017 - 20:55 trong Thi HSG Quốc gia và Quốc tế

Bài 2:

Ta sẽ chứng minh rằng số hoán vị chứa đúng $k$ $A-inversion$ không phụ thuộc vào dãy $A$ (Và bằng số số hoán vị có đúng $k$ nghịch thế).

Bài toán hiển nhiên đúng với $n=1$, giả sử bài toán đúng với $n=k$, xét $n=k+1$. Xét $1$ hoán vị có chứa số $w_1$ đứng đầu dãy. Gọi $f$ là số số $w_i$ thoả mãn $(w_1;w_j)$ là $1$ $A-inversion$. Nếu $a_1>w_1$, số số $w_j$ thoả mãn điều kiện bằng số số $t$ trong $n$ số $m_1,m_2,...,m_n$ thoả mãn $t>a_1$ hoặc $t<w_1$, số các số đó hoàn toàn không phụ thuộc vào cách sắp xếp $n-1$ số cồn lại khác $w_1$. Tương tự với trường hợp $a_1<w_1;a_1=w_1$. Gọi số số $w_j$ lạp thành cặp $A-inversion$ với $w_1$ là $f$, ta đếm cách sắp xếp $n-1$ số còn lại để có đúng $k-f$ cặp, và nó không phụ thuộc vào $A-a_1$. Tương tự với các trường hợp những số khác xếp ở vị trí đầu trong hoán vị, tổng lại các cách sắp xếp thì ta được số cách sắp xếp để có đúng $k$ cặp $A-inversion$ và nó không phụ thuộc vào $A$. Mệnh đề đúng với $n=k+1$, $\Rightarrow$ $Q.E.D$




#675516 ĐỀ VIỆT NAM TST 2017

Đã gửi bởi JUV on 28-03-2017 - 13:24 trong Thi HSG Quốc gia và Quốc tế

 

 

Chắc là không, vì nếu mà như dungxibo123 nói thì khi đó sẽ có trường hợp tất cả các con kiến đều gặp nhau khi cùng xuất phát tại 1 lỗ chung, dẫn đến bài toán sai.

Nếu như cả $2017$ con đều lên từ một lỗ thì không có 2 con nào xuống cùng 1 lỗ tại cùng 1 thời điểm. Tại mỗi thời điểm sẽ có $43$ lỗ để xuống nên có nhiều nhất $43$ con xuống lỗ. Tuy nhiên vì có $2017$ con đã lên nên số lần xuống ít nhất là $\frac{2017}{43}>45$




#675457 Một bài tổ hợp từ một bài số học

Đã gửi bởi JUV on 27-03-2017 - 18:00 trong Tổ hợp và rời rạc

định nghĩa một xích là $1$ dãy các tập hợp $(A_{a_1};A_{a_2};...;A_{a_t})$ thoả mãn $0<a_1<a_2<...<a_t<T+1$ và $A_{a_i}\subset A_{a_{i+1}} \forall 1\leq i\leq t-1$. Với mỗi tập $A_i$, gọi $f(A_i)$ là xích dài nhất nhận $A_i$ là tập đầu tiên trong xích. Giả sử mọi xích đều có độ dài $\leq n-1$ thì lúc đó tồn tại $n$ tập $A_{b_i} \forall i=\overline{1,n}$ thoả mãn $f(A_{b_i})=f(A_{b_j}) \forall i,j=\overline{1,n}$. Nếu tồn tại $i;j$: $1\leq i< j\leq n$ để $A_{b_i}\subset A_{b_j}$ thì $f(A_{b_i})\geq f(A_{b_j})+1$ (vô lí). Vì vậy $n$ tập trên không thoả mãn đề bài, suy ra giả sử sai. Vậy tồn tại $1$ xích độ dài $n$, nhận $A_t$ làm tập cuối cùng trong dãy. Lúc đó $\left | A_t \right |\geq n\Rightarrow A_t=A$.




#674526 Mở rộng bài T3 Tạp chí Toán học và tuổi trẻ số 477 tháng 1 2017

Đã gửi bởi JUV on 17-03-2017 - 14:16 trong Bất đẳng thức và cực trị

Tất cả các vấn đề đều có thể được giải quyết bằng BĐT hoán vị




#671607 [Trường Xuân toán học miền nam 2016] Vietnam TST 2016 MOCK Test 2

Đã gửi bởi JUV on 14-02-2017 - 13:57 trong Thi HSG cấp Tỉnh, Thành phố. Olympic 30-4. Đề thi và kiểm tra đội tuyển các cấp.

Đào mộ !

Bài 4:

Nếu $m,n$ cùng chia hết cho $3$ thì đáp số là $mn$

Nếu $m$ chia hết cho $3$ còn $n$ thì không, đáp số là $m(n-1)$(Nếu có $m(n-1)+1$ số $1$ thì trong $m$ hàng sẽ có $1$ hàng chứa $n$ số $1$, tổng $n$ số đó không chia hết cho $3$)

Nếu $m \equiv n\not\equiv 0 \pmod{3}$ thì đáp số là $mn-max\left \{ m;n \right \}$, lập luận tương tự trường hợp trên.

Nếu $m \equiv n+1\equiv 2 \pmod{3}$, ta chứng minh sẽ có ít nhất $\frac{2}{3}(m+n)$ ô không chứa số $1$. Gọi các ô không chứa số $1$ là ô lạ, dễ thấy rằng không có ô lạ nào đứng một mình, tức là không có ô lạ nào khác đứng cùng hàng hoặc cột với nó (Nếu không thì tổng trên cột và hàng chứa ô lạ đó sẽ hơn kém nhau $1$ nên không thể cùng chia hết cho $3$). Ta sẽ tô màu các ô của bảng theo cách sau: Đầu tiên chọn $1$ ô lạ, tô màu tất cả các ô chưa được tô màu nằm cùng hàng và cột với ô đó, sau đó chọn $1$ ô lạ khác chưa được chọn và tiếp tục. Dễ thấy mỗi hàng và cột phải có ít nhất $1$ ô lạ nên rút cuộc tất cả các ô trên bảng đều được tô, hay $m+n$ hàng và cột sẽ được tô(một hàng hoặc cột được tô khi tất cả các ô trên nó được tô). Khi chọn $1$ ô lạ bất kì thì tô được thêm nhiều nhất $2$ đường ($1$ đường tức là $1$ hàng hoặc cột). Nếu sau khi chọn $1$ ô lạ ta tộ được $2$ đường chứa nó thì sẽ có $1$ ô lạ khác nằm cùng hàng hoặc cột với nó chưa được chọn (Nếu được chọn trước rồi thì sau khi chọn ô lạ kia, ta chỉ có thể tô thêm được $1$ đường qua nó). Vậy có cách chọn để cứ chọn $2$ ô lạ liên tiếp, ta tô thêm được nhiều nhất $3$ đường $\Rightarrow$ Số ô lạ ít nhất là $\frac{2}{3}(m+n)$. Vậy số ô có chứa số $1$ ít nhất là $mn-\frac{2}{3}(m+n)$. Lưu ý rằng nếu $max\left \{ m;n \right \}>2min\left \{ m;n \right \}$ thì $max\left \{ m;n \right \}>\frac{2}{3}(m+n)$, lập luận tương tự các tường hợp trên thì số số $1$ nhiều nhất là $mn-max\left \{ m;n;\frac{2}{3}(m+n) \right \}$. Dấu "=" xảy ra khi:

Với $max\left \{ m;n \right \}\leq 2min\left \{ m;n \right \}$(giả sử $n>m$), chọn $\frac{2n-m}{3}$ hàng, mỗi hàng điền $2$ số $0$ sao cho không có $2$ số $0$ nào cùng cột. Trong $\frac{2m-n}{3}$ cột không có số $0$, điền vào mỗi cột $2$ số $1$ sao cho không có $2$ số $1$ hoặc $2$ số $0$ và$1$ cùng hàng.

Nếu $n>2m$, từ $1$ bảng $m\times 2m$ được điền, ta chọn thêm $n-2m$ cột và điền vào mỗi cột đó số $0$ vào ô ở dưới cùng và số $1$ vào các ô còn lại




#670226 $x^{2}+y^{2}=n$

Đã gửi bởi JUV on 28-01-2017 - 15:33 trong Số học

Tất cả các số $n$ thoả mãn $v_p(n)\vdots 2$ với mọi $p$ nguyên tố dạng $4k+3$. Lưu ý rằng mọi số nguyên tố có dạng $4k+1$ đều biểu diễn được dưới dạng $x^2+y^2$ và nếu $a,b$ biểu diễn được thì $ab$ cũng biểu diễn được ($(x^2+y^2)(m^2+n^2)=(xm+yn)^2+(xn-ym)^2$). Ngược lại, dễ chứng minh nếu $n=x^2+y^2$ chia hết cho $p$ với $p$ nguyên tố dạng $4k+3$ thì cả $x,y$ đều chia hết cho $p$, từ đó tổng bình phương chia hết cho $p^2$ nên $v_p(n)\vdots 2$.




#669702 Đề Thi VMO năm 2017

Đã gửi bởi JUV on 24-01-2017 - 15:17 trong Thi HSG Quốc gia và Quốc tế

File kết quả cho mọi người, điểm TST là $23$ nhé :File gửi kèm  in_dsdoatgiai_HSGQG_2017-1.pdf   1.05MB   377 Số lần tải




#665012 2012 ELMO Shortlist C1

Đã gửi bởi JUV on 18-12-2016 - 17:16 trong Tổ hợp và rời rạc

Ta sẽ chứng minh bằng quy nạp rằng $m=2n+1$ là giá trị nhỏ nhất cần tìm. Dễ thấy mệnh đề đúng với $n=2$, giả sử mệnh đề đúng với $n=k$. Xét dãy gồm $m$ số $a_1,a_2,...,a_m$ thoả mãn với mọi lớp có thể có độ dài $k+1$, tồn tại một dãy con của dãy đó có lớp như thế. Giả sử $a_1=a_2$, ta xét các lớp có dạng $(-1;i_1;i_2;...;i_k)$ với $i_t\in \left \{ -1;1 \right \}\forall t=\overline{1,k}$ và $a_{t_1};a_{t_2};...;a_{t_{k+2}}$ là dãy có lớp như thế với $t_1<t_2<...<t_{k+2}$. Có $a_{t_1}\neq a_{t_2}$ nên $t_2>2$. Cũng có dãy $a_{t_2};a_{t_3};...;a_{t_{k+2}}$ là một dãy con có lớp $(i_1;i_2;...;i_k)$ và dãy đó là một dãy con của dãy $a_3,a_4,...,a_m$ nên khi cho các số $i_t$ thay đổi tạo thành tất cả các lớp có thể có độ dài $k$ thì dãy $a_3,a_4,...,a_m$ luôn có một dãy con có lớp như thế. Dãy trên gồm $m-2$ số nên theo giả thiết quy nạp thì $m-2\geq 2k+1$ nên $m\geq 2k+3$. Nếu $a_1\neq a_2$ thì lập luận tương tự với lớp độ dài $k+1$ có dạng $(1;i_1;i_2;...;i_k)$ với $i_t\in \left \{ -1;1 \right \}\forall t=\overline{1,k}$, cũng có $m\geq 2k+3$. Vậy giả thiết quy nạp đúng, hay $m\geq 2n+1$. Có thể xét dãy $2n+1$ số sau: $1,0,1,0,1,0,...,0,1$($n+1$ số $1$ và $n$ số $0$ xếp xen kẽ).




#664542 $m\leq 2^{n-1}-1$

Đã gửi bởi JUV on 13-12-2016 - 16:04 trong Tổ hợp và rời rạc

Ta sẽ quy nạp theo $n$, dễ thấy $n=2$ đúng. Giả sử bài toán đúng với $n=k$. Xét $S= \left \{ 1;2;...;k+1 \right \}$. Xét các cặp $(A;S\setminus A)$ với $A\in S, \left | A \right |\geq 2$, nếu tồn tại $M,N$ trong $m$ tập thuộc cùng $1$ cặp thì xét $A_i$ $\forall i=\overline{1,m}$, luôn tồn tại $B_i\in M$ để $A_i\cap M=B_i$. $\forall T\subset M,T\neq \varnothing,T\neq M$, gọi $d_T$ là số các tập $A_i$ để $A_i\cap M=T$ và $A_i\neq T$. Nếu $\exists A_i,A_j: A_i\cap N=A_j\cap N\neq \varnothing, A_i\cap M=T,A_j\cap M=M\setminus T$ thì $3$ tập $(A_i,A_j,M)$ không thoả mãn đề bài. Vậy $A_i\cap N\neq A_j\cap N$ $\forall A_i,A_j:A_i\cap M=T=M\setminus (A_j\cap M)$ $(A_i\neq T,A_j\neq M\setminus T)$. $N$ có $2^{\left | N \right |}-1$ tập con khác rỗng nên $d_T+d_{M\setminus T}\leq 2^{\left | N \right |}-1$.Với $T= \varnothing$, $d_{\varnothing}\leq 2^{\left | N \right |-1}-1$ theo giả thuyết quy nạp, mặt khác với $A_i,A_j$ chứa $M$ và khác $M$ thì $(A_i\cap N)\cap (A_j\cap N)\neq \varnothing$, nếu không thì $(A_i,A_j,N)$ là $3$ tập không thoả mãn. Từ đó suy ra $d_M\leq 2^{\left | N \right |-1}$, vậy $d_M+d_{\varnothing}\leq 2^{\left | N \right |}-1$. Vậy $d_T+d_{M\setminus T}\leq 2{^\left | N \right |}-1\forall T\subset M$ .Gọi $p$ là số tập con $S$ trong $m$ tập mà là tập con của $M$, vì $\left | M \right |<k+1$ nên theo giả thiết quy nạp thì $p\leq 2^{\left | M \right |-1}-1$. Vậy $m=\frac{\sum_{T\subset M}(d_T+d_{M\setminus T})}{2}+p\leq (2^{\left | N \right |}-1)2^{\left | M \right |-1}+2^{\left | M \right |-1}-1=2^{\left | M \right |+\left | N \right |-1}-1= 2^k-1$. Nếu không tồn tại $M,N$ thuộc cùng $1$ cặp, lúc đó $m\leq 2^{n-1}$. Nếu $m=2^{n-1}$, mỗi cặp chỉ có $1$ tập con được chọn. Dễ thấy tất cả tập con có số phần tử bằng $k$ và $k+1$ được chọn, có thể dễ dàng chứng minh nếu tất cả tập con có $i+1$ phần tử được chọn thì tất cả tập con có $i$ phần tử cũng được chọn với $i>\frac{k}{2}$. Vậy tất cả các tập con có $>\frac{k}{2}$ phần tử đều được chọn, dễ suy ra điều vô lí. Vây $m\leq 2^{n-1}-1$ 




#664345 Chứng minh tồn tại hình chữ nhật có các đỉnh cùng màu.

Đã gửi bởi JUV on 11-12-2016 - 08:38 trong Tổ hợp và rời rạc

Tồn tại $1$ màu sao cho màu đó tô ít nhất $40$ điểm. Gọi $a_1,a_2,...,a_{12}$ là số điểm được tô màu đó trên $12$ đường $x=i$ với $i=\overline{1,12}$. Số cặp điểm cùng màu đã chọn là $\binom{a_i}{2}=\frac{(a_i-1)a_i}{2}$ $\forall i=\overline{1,12}$. Tổng trên cả bảng có $\frac{\sum_{i=1}^{12}a_i^2-a_i}{2}\geq \frac{\frac{40^2}{12}-40}{2}> 46$ cặp điểm cùng màu được chọn và cùng cột. Có $\binom{10}{2}= 45$ cách chọn ra $2$ số nguyên khác nhau thuộc đoạn $\left [ 1;10 \right ]$ nên tồn tại $2$ cặp điểm cùng màu được chọn và cùng cột $(A_1,B_1),(A_2,B_2)$ sao cho tung độ $A_1=A_2$, tung độ $B_1=B_2$ . $A_1B_1B_2A_2$ là hình chữ nhật cần tìm.